0 Daumen
779 Aufrufe

Ich sitzt jetzt schon ziemlich lange an dieser Aufgabe aber ich komm einfach nicht auf die richtige Lösung..

Die Nutzenfunktion eines Individuums lautet U ( x1 , x2 ) = x10.6 * x2 0.8 . Gegeben sind die Preise der beiden Güter p1 =1 und p2 =5 sowie das zur Verfügung stehende Einkommen in Höhe von I=130. Optimieren Sie den Nutzen des Individuums unter Beachtung seiner Budgetrestriktion.


Wie hoch ist die Menge x2 in diesem Nutzenoptimum?

Ich hab also die Lagrange-Funktion aufgestellt:  x1 0,6 * x20,8 - lamda(1x 1 + 5x2 - 130)

dann das einmal nach x1 aufgelöst => 0,6-130*lamda*5x2

nach x2 => 0,8 - 130*lamda*1x1

und dann nach lamda => 130-1x1^0,6 * 5x2^0,8

 

aber schon da bin ich mir nicht sicher ob das stimmt.. könnte mir mal bitte jemand helfen? (:

Avatar von

1 Antwort

0 Daumen

Unbenannt.PNG

Text erkannt:

\( U(x, y)=x^{0,6} \cdot y^{0,8} \)
\( x+5 y=130 \)
\( U(x, y, \lambda)=x^{\frac{3}{3}} \cdot y^{\frac{4}{3}}+\lambda \cdot(x+5 y-130) \)
\( \frac{d U(x, y, \lambda)}{d x}=\frac{\frac{3}{5}}{x^{\frac{2}{3}}} \cdot y^{\frac{4}{3}}+\lambda \)
\( \frac{d U(x, y, \lambda)}{d y}=x^{\frac{3}{5}} \cdot \frac{\frac{4}{5}}{y^{\frac{1}{5}}}+5 \lambda \)
\( \frac{d U(x, y, \lambda)}{d \lambda}=x+5 y-130 \)
1. \( \left|\frac{\frac{3}{5}}{x^{\frac{2}{5}}}-y^{\frac{4}{5}}+\lambda=0\right| \cdot 5 \)
1. \( ) \frac{3}{x^{\frac{2}{3}}} \cdot y^{\frac{4}{5}}+5 x=0 \)
\( x^{\frac{3}{5}} \cdot \frac{\frac{4}{5}}{y^{\frac{1}{5}}}+5 \lambda=0 \)
\( \frac{3 \cdot y^{4}}{x^{\frac{2}{5}}}=x^{\frac{8}{5}} \cdot \frac{\frac{4}{5}}{y^{\frac{1}{5}}} \)
\( 3 y=\frac{4}{5} x \)
\( y=\frac{4}{15} x \)
\( x+\frac{4}{3} x-130=0 \)
\( x=\frac{390}{7} \)
\( y=\frac{4}{15} \cdot \frac{390}{7}=\frac{104}{7} \)
\( \frac{\frac{3}{5}}{\left(\frac{390}{7}\right)^{\frac{2}{5}}} \cdot\left(\frac{104}{7}\right)^{\frac{4}{5}}+\lambda=0 \)
\( 1 \times 1,04 \)
\( U=\left(\frac{390}{7}\right)^{0.6} \cdot\left(\frac{104}{7}\right)^{0,8} \approx 96,6 \mathrm{mfG} \) Moliets

Avatar von 36 k

Ein anderes Problem?

Stell deine Frage

Willkommen bei der Mathelounge! Stell deine Frage einfach und kostenlos

x
Made by a lovely community